Is the Series Convergent or Divergent?

  • Thread starter Thread starter Ethan Godden
  • Start date Start date
  • Tags Tags
    Limits Series
Click For Summary
The discussion revolves around determining the convergence or divergence of a summation using the Alternating Series Test and the Test for Divergence. Participants note that while the limit of the terms approaches zero, the alternating nature of the series complicates the application of the tests. There is confusion regarding the expression ##(-1)^\infty##, which is deemed undefined and leads to questions about the validity of the divergence test. Ultimately, it is clarified that the terms of the series decrease in magnitude, allowing the series to converge despite the alternating signs. The key takeaway is that the limit approaching zero is crucial for convergence in alternating series.
Ethan Godden
Messages
33
Reaction score
0

Homework Statement


I am supposed to determine whether the summation attached is convergent or divergent

Homework Equations


Alternating Series Test
Test for Divergence

The Attempt at a Solution


The attempted solution is attached. Using the two different tests I am getting two different answers.
 

Attachments

Physics news on Phys.org
It is much preferred for you to type the problems rather than post a download.

You have ##\frac 1 {\sqrt{n+1}}\to 0## which is correct. Now since$$
0 \le \left | \frac {(-1)^n} {\sqrt{n+1}}\right | \le \frac 1 {\sqrt{n+1}}$$ how could the alternating one not go to zero? And, by the way, ##(-1)^\infty## makes no sense.
 
Last edited:
Okay, you used the squeeze theorem which makes sense, but why doesn't the test for divergence work? Isn't (-1) undefined meaning the limit is undefined meaning the series is divergent?
 
Ethan Godden said:
Okay, you used the squeeze theorem which makes sense, but why doesn't the test for divergence work? Isn't (-1) undefined meaning the limit is undefined meaning the series is divergent?

Yes, as I said, ##(-1)^\infty## makes no sense or, as you say, is undefined. What is happening in this problem is that the denominator is getting larger and the numerator is either plus or minus 1 for any n. The fraction gets small no matter the sign, so regardless of the alternating sign the fraction goes to zero.
 
Question: A clock's minute hand has length 4 and its hour hand has length 3. What is the distance between the tips at the moment when it is increasing most rapidly?(Putnam Exam Question) Answer: Making assumption that both the hands moves at constant angular velocities, the answer is ## \sqrt{7} .## But don't you think this assumption is somewhat doubtful and wrong?

Similar threads

  • · Replies 4 ·
Replies
4
Views
1K
  • · Replies 4 ·
Replies
4
Views
2K
  • · Replies 3 ·
Replies
3
Views
1K
  • · Replies 2 ·
Replies
2
Views
2K
  • · Replies 6 ·
Replies
6
Views
2K
  • · Replies 4 ·
Replies
4
Views
1K
  • · Replies 5 ·
Replies
5
Views
2K
  • · Replies 2 ·
Replies
2
Views
1K
  • · Replies 8 ·
Replies
8
Views
2K
  • · Replies 7 ·
Replies
7
Views
2K